www.vorkurse.de
Ein Projekt von vorhilfe.de
Die Online-Kurse der Vorhilfe

E-Learning leicht gemacht.
Hallo Gast!einloggen | registrieren ]
Startseite · Mitglieder · Teams · Forum · Wissen · Kurse · Impressum
Forenbaum
^ Forenbaum
Status Mathe-Vorkurse
  Status Organisatorisches
  Status Schule
    Status Wiederholung Algebra
    Status Einführung Analysis
    Status Einführung Analytisc
    Status VK 21: Mathematik 6.
    Status VK 37: Kurvendiskussionen
    Status VK Abivorbereitungen
  Status Universität
    Status Lerngruppe LinAlg
    Status VK 13 Analysis I FH
    Status Algebra 2006
    Status VK 22: Algebra 2007
    Status GruMiHH 06
    Status VK 58: Algebra 1
    Status VK 59: Lineare Algebra
    Status VK 60: Analysis
    Status Wahrscheinlichkeitst

Gezeigt werden alle Foren bis zur Tiefe 2

Navigation
 Startseite...
 Neuerdings beta neu
 Forum...
 vorwissen...
 vorkurse...
 Werkzeuge...
 Nachhilfevermittlung beta...
 Online-Spiele beta
 Suchen
 Verein...
 Impressum
Das Projekt
Server und Internetanbindung werden durch Spenden finanziert.
Organisiert wird das Projekt von unserem Koordinatorenteam.
Hunderte Mitglieder helfen ehrenamtlich in unseren moderierten Foren.
Anbieter der Seite ist der gemeinnützige Verein "Vorhilfe.de e.V.".
Partnerseiten
Weitere Fächer:

Open Source FunktionenplotterFunkyPlot: Kostenloser und quelloffener Funktionenplotter für Linux und andere Betriebssysteme
Forum "Partielle Differentialgleichungen" - Loesung PDE
Loesung PDE < partielle < Differentialgl. < Analysis < Hochschule < Mathe < Vorhilfe
Ansicht: [ geschachtelt ] | ^ Forum "Partielle Differentialgleichungen"  | ^^ Alle Foren  | ^ Forenbaum  | Materialien

Loesung PDE: Frage (überfällig)
Status: (Frage) überfällig Status 
Datum: 15:05 So 27.08.2017
Autor: Chris84

Huhu,
ich habe mal wieder eine Frage:

Ich habe die inhomogene partielle Differentialgleichung

$ [mm] \frac{m_e}{n_e q_e}\frac{1}{4\pi c}\frac{\partial}{\partial t}\Delta \vec{C}(\vec{r},t)-\frac{m_e}{n_e q_e}\frac{1}{4\pi c^3}\frac{\partial^3}{\partial t^3}\vec{C}(\vec{r},t)-q_m \vec{\nabla}\times\vec{C}(\vec{r},t)=\vec{S}(\vec{r},t)$, [/mm]

wobei [mm] $\Delta$ [/mm] der Laplaceoperator, [mm] $\vec{S}$ [/mm] eine Stoerfunktion und [mm] $\vec{C}$ [/mm] die gesuchte Funktion sind.

Die homogene Loesung ist einfach zu berechnen; sie lautet

[mm] $\vec{C}_{hom}(\vec{r},t)=\vec{a}\cdot e^{\vec{k}\cdot\vec{r}-\omega\cdot t}$ [/mm]
mit [mm] $\vec{a}\perp\vec{k}$ [/mm] und [mm] $k^2=\omega^2/c^2$. [/mm]

Die Problem ist nun die partikulaere Loesung. Mein Ansatz war der ueber die Greenschen Funktionen. Fuer die Greensche Funktion [mm] $\vec{G}(\vec{r},t)$ [/mm] lautet dann die entsprechende PDE:

$ [mm] \frac{m_e}{n_e q_e}\frac{1}{4\pi c}\frac{\partial}{\partial t}\Delta \vec{G}(\vec{r},t)-\frac{m_e}{n_e q_e}\frac{1}{4\pi c^3}\frac{\partial^3}{\partial t^3}\vec{G}(\vec{r},t)-q_m \vec{\nabla}\times\vec{G}(\vec{r},t)=\delta(\vec{r})\delta(t)\vec{a}$ [/mm]

wobei [mm] $\delta(\cdot)$ [/mm] die Diracsche Deltedistribution bezeichne.

So, aber diese PDE ist nicht einfach zu loesen. Meine erste Idee, war in den vierdimensionalen Fourierraum zu wechseln. Mit

[mm] $\delta(t)=\frac{1}{2\pi}\int\limits_{-\infty}^{\infty} e^{i\omega t} d\omega [/mm]
[mm] \delta(t)=\frac{1}{(2\pi)^3}\int\limits_{\IR^3} e^{i\vec{k}\cdot\vec{r}} d^3\vec{k} [/mm]
[mm] \vec{G}(\vec{r},t)=\frac{1}{(2\pi)^4}\int\limits_{\IR^4} \hat{\vec{G}}(\vec{k},\omega) e^{i\vec{k}\cdot\vec{r}+i\omega t} d^3\vec{k} d\omega$ [/mm]

und [mm] $\alpha:=frac{m_e}{n_e q_e}\frac{1}{4\pi c}$ [/mm] bekommt man die Gleichung

[mm] $-i\omega k^2\alpha \hat{\vec{G}}+i\omega^3\frac{\alpha}{c^2} \hat{\vec{G}}-q_m [/mm] i [mm] \vec{k}\times\hat{\vec{G}}=\vec{a}$ [/mm] (*).

Die Loesung dieser algebraischen Gleichung ist

[mm] $\hat{\vec{G}}(\vec{k},\omega)=\frac{(-i\omega k^2\alpha+i\omega^3\frac{\alpha}{c^2})\vec{a}}{(-i\omega k^2\alpha+i\omega^3\frac{\alpha}{c^2})-q_m^2 k^2}-\frac{q_m^2(\vec{k}\cdot\vec{a})\vec{k}}{(-i\omega k^2\alpha+i\omega^3\frac{\alpha}{c^2})^3-(-i\omega k^2\alpha+i\omega^3\frac{\alpha}{c^2})q_m^2 k^2}+\frac{i q_m\vec{k}\times\vec{a}}{(-i\omega k^2\alpha+i\omega^3\frac{\alpha}{c^2})^2-q_m^2k^2}$. [/mm]

Das Problem ist nun diese Loesung zurueck zu transformieren. Da man fuer den ersten Term Kugelkoordinaten fuer [mm] $\vec{k}$ [/mm] benutzen kann, habe ich mich mal daran versucht. Am Ende bleibt das Integral

[mm] $\int\limits_{-\infty}^{\infty} d\omega \omega^2 e^{-\frac{q_m}{2\omega\alpha}r}\left[\frac{2\left(\frac{q_m}{2\omega\alpha}\right)^2}{\sqrt{-\left(\frac{q_m}{2\omega\alpha}\right)^2+\frac{\omega^2}{c^2}}}\sin\left(\sqrt{-\left(\frac{q_m}{2\omega\alpha}\right)^2+\frac{\omega^2}{c^2}} r\right)-\frac{2q_m}{\omega\alpha}\cos\left(\sqrt{-\left(\frac{q_m}{2\omega\alpha}\right)^2+\frac{\omega^2}{c^2}} r\right)\right]e^{i\omega t}$. [/mm]

Mal davon abgesehen, dass ich keine Idee habe, wie man dieses Monstrum berechnen soll, scheint es, dass das Integral nicht 'mal konvergent ist.

Meine naechste Idee war, (*) nur in [mm] $\vec{r}$ [/mm] zu transformieren (um die Ruecktransformation zu vereinfachen). Dies fuehrt zur gewoehnlichen Differentialgleichung

$ [mm] \frac{m_e}{n_e q_e}\frac{1}{4\pi c}\frac{\partial}{\partial t} (-k^2) \hat{\vec{G}}(\vec{k},t)-\frac{m_e}{n_e q_e}\frac{1}{4\pi c^3}\frac{\partial^3}{\partial t^3}\hat{\vec{G}}(\vec{k},t)-q_m i\vec{k}\times\hat{\vec{G}}(\vec{k},t)=\delta(t)\vec{a}$ [/mm] (**).

Wenn ich diese ODE loesen koennte, muesste ich nur in [mm] $\vec{k}$ [/mm] zuruecktransformieren. Die homogene Loesung zu bekommen, ist auch hier nicht sonderlich schwierig. Allerdings scheitere ich auch hier wieder an der partikulaeren Loesung (Variation der Konstanten habe ich versucht, liefert nur leider keine Loesung).

Hat vlt. jemand eine Idee, wie man die ODE (**) oder die urspruengliche PDE (*) loesen koennte.

Gruss,
Chris

P.S.: Es scheint, dass der Latexeditor gerade nicht funktioniert. Das macht das Kontrollieren der eingegeben Formeln nicht gerade einfach. Ich bitte um Nachsicht.

        
Bezug
Loesung PDE: Fälligkeit abgelaufen
Status: (Mitteilung) Reaktion unnötig Status 
Datum: 15:20 Mi 27.09.2017
Autor: matux

$MATUXTEXT(ueberfaellige_frage)
Bezug
        
Bezug
Loesung PDE: Mitteilung
Status: (Mitteilung) Reaktion unnötig Status 
Datum: 15:30 Mi 27.09.2017
Autor: Martinius

Hallo Chris84,

Du könntest auch hier:

[]http://matheplanet.org/

fragen.

LG, Martinius

Bezug
Ansicht: [ geschachtelt ] | ^ Forum "Partielle Differentialgleichungen"  | ^^ Alle Foren  | ^ Forenbaum  | Materialien


^ Seitenanfang ^
www.vorkurse.de
[ Startseite | Mitglieder | Teams | Forum | Wissen | Kurse | Impressum ]